Can someone please help me?

Can Someone Please Help Me?

Answers

Answer 1

Answer:

use the Desmos graphing calculator.  Type the first one in.  Then type your answer choices in.  See which lines are the same.

Step-by-step explanation:

Answer 2

Answer:

D

Step-by-step explanation:

Just simplify


Related Questions

You earn $36 for washing 6 cars. How much do you earn for washing 3 cars?

Answers

Answer:

18$

Step-by-step explanation:

36 divided by 6 is 6 dollars. this means you earn 6 dollars per car. multiply that by 3 cars and you get 18$.

Answer:

You earn $18 for washing 3 cars.

$36➗ 6= $6

$6x3= $18

Simplify the expression

Answers

Answer:

6

Step-by-step explanation:

3 sqrt(20) / sqrt(5)

We know that sqrt(a) /sqrt(b) = sqrt(a/b)

3 sqrt(20/5)

3 sqrt(4)

3 *2

6

what the person above me said

Which shows the following expression after the negative exponents have been eliminated?

Answers

Step-by-step explanation:

The given expression is :

[tex]\dfrac{a^3b^{-2}}{ab^{-4}}[/tex]

We need to simplify the above expression.

a³ is in numerator and a is in denominator. It gts cancelled.

[tex]\dfrac{a^3b^{-2}}{ab^{-4}}=\dfrac{a\times a\times a\times b\times b\times b\times b}{a\times b \times b}\\\\=\dfrac{a^2\times b^{-2}\times b^4}{1}\\\\=\dfrac{a^2}{b^{-2}}[/tex]

Hence, this is the required solution.

PLEASE HELP ME WILL MARK YOU JF YOU HELP ME PLEASE!!!

Answers

Answer:

2, 3, 4, 7, 8, 10 I hope

Answers

Congruent by AAS (shown in the example)Congruent by SASCongruent by SSSCongruent by ASANot enough info  (shown in the second example)Congruent by AASCongruent by SASCongruent by SSSCongruent by AASNot congruent

=================================================

Explanations:

As the example shows, we have two pairs of congruent angles and a pair of congruent sides. The side are not between the angles in question. So AAS is slightly different from ASA.We have two pairs of congruent sides, and a pair of congruent angles. The angles are between the sides. So we use SAS which is a valid congruence theorem. Recall that SSA is not a valid theorem, so the order matters.We have three pairs of congruent sides, so we go with SSS. The order doesn't matter here.Similar to problem 1, but now the sides are between the angles. So we go with ASA this time instead of AAS.We unfortunately don't have enough info to determine if the triangles are congruent or not. We need to know something about the side lengths to determine congruency.As the hint suggests, marking the vertical angles will produce the other pair of congruent angles. So that's why we go for AAS (the side is not between the angles).This is similar to problem 2, as both use SAS. Note the unmarked vertical angles which are congruent.This is similar to problem 3. We use SSS here because we have 3 pairs of congruent sides as indicated by the tickmarks.The unmarked vertical angles can get double arcs to show they are congruent. We have a pair of congruent sides that are not between the two pairs of congruent angles, so we go for AAS (problems 1 and 6 also use AAS).For the triangle on the left, the arc is between the tickmarked sides. The triangle on the right has the arc not between the tickmarked sides. So there's no way the triangles are the same. The arc needs to be between the marked sides for each triangle, if we wanted them to be congruent (using SAS).

---------------

Acronyms

SSS = side side side

SAS = side angle side

ASA = angle side angle

AAS = angle angle side

Write the equation of the line that passes through the points (8,-1) and (2,-5) in standard from, given that the point-slope form is y+1=2/3(x-8)

____x+____y=____

Answers

Answer: 2/3x-y=19/3

Step-by-step explanation:

Points are useless since we already know the point slope form and we can just simply that

y+1=2/3(x-8)

y+1=2/3x-16/3

y=2/3x-19/3

2/3x-y=19/3

For each of the following, shade the portion of the Venn diagram that illustrates the set. ( A ∩ B ) ∪ ( A ∩ C )

Answers

Answer:

Here's what that would look like on a venn diagram

Step-by-step explanation:

Find the value of x and show work

Answers

Answer:

20

Step-by-step explanation:

Sketch the region enclosed by the given curves and calculate its area.
y=4-x^2 ,y=0

The answer is 32/3. But how do I get to that answer?

Answers

Answer:

Step-by-step explanation:

1.) we need to find the bounds of integration which is just the points of intersection

here is it (-2,0) and (2,0)

which means we will integrate from -2 to 2

next, we take the upper equation and subtract that from the lower one

kind of confusing but it would look like (sketch it out if you're not sure)

(4-x²)-0= 4-x²

then we can integrate

[tex]\int\limits^2_{-2} {4-x^2} \, dx =4x-\frac{x^3}{3}|_{-2}^{2}=(4*(2)-\frac{2^3}{3})-(4(-2)-\frac{-2^3}{3})=5.333333-(-5.3333333)= 10.666666667=\frac{32}{3}[/tex]

There are twelve empty rooms in the office of Acme Softwares, inc. Alif and Laila joins the company. How many ways can they be assigned a room each from these twelve

Answers

Answer:

132.

Step-by-step explanation:

That would be the number of permutations of 2 from 12.

12P2 = 12! / (12-2)!

= 132.

Differentiate the function. y = (2x - 5)^2 (5 - x)?​

Answers

Answer:

[tex]\displaystyle y' = -(2x - 5)(6x - 25)[/tex]

General Formulas and Concepts:

Pre-Algebra

Order of Operations: BPEMDAS

Brackets Parenthesis Exponents Multiplication Division Addition Subtraction Left to Right

Distributive Property

Algebra I

Terms/CoefficientsFactoring

Calculus

Derivatives

Derivative Notation

Derivative of a constant is 0

Basic Power Rule:

f(x) = cxⁿ f’(x) = c·nxⁿ⁻¹

Derivative Rule [Product Rule]:                                                                                    [tex]\displaystyle \frac{d}{dx} [f(x)g(x)]=f'(x)g(x) + g'(x)f(x)[/tex]

Derivative Rule [Chain Rule]:                                                                                       [tex]\displaystyle \frac{d}{dx}[f(g(x))] =f'(g(x)) \cdot g'(x)[/tex]

Step-by-step explanation:

Step 1: Define

Identify

y = (2x - 5)²(5 - x)

Step 2: Differentiate

Derivative Rule [Product Rule]:                                                                        [tex]\displaystyle y' = \frac{d}{dx}[(2x - 5)^2](5 - x) + (2x - 5)^2\frac{d}{dx}[(5 - x)][/tex]Chain Rule [Basic Power Rule]:                                                                        [tex]\displaystyle y' = [2(2x - 5)^{2 - 1} \cdot \frac{d}{dx}[2x]](5 - x) + (2x - 5)^2\frac{d}{dx}[(5 - x)][/tex]Simplify:                                                                                                             [tex]\displaystyle y' = [2(2x - 5) \cdot \frac{d}{dx}[2x]](5 - x) + (2x - 5)^2\frac{d}{dx}[(5 - x)][/tex]Basic Power Rule:                                                                                             [tex]\displaystyle y' = [2(2x - 5) \cdot 1 \cdot 2x^{1 - 1}](5 - x) + (2x - 5)^2(1 \cdot -x^{1 - 1})][/tex]Simplify:                                                                                                             [tex]\displaystyle y' = [2(2x - 5) \cdot 2](5 - x) + (2x - 5)^2(-1)[/tex]Multiply:                                                                                                             [tex]\displaystyle y' = 4(2x - 5)(5 - x) - (2x - 5)^2[/tex]Factor:                                                                                                               [tex]\displaystyle y' = (2x - 5)[4(5 - x) - (2x - 5)][/tex][Distributive Property] Distribute 4:                                                                 [tex]\displaystyle y' = (2x - 5)[20 - 4x - (2x - 5)][/tex][Distributive Property] Distribute negative:                                                    [tex]\displaystyle y' = (2x - 5)[20 - 4x - 2x + 5][/tex][Subtraction] Combine like terms (x):                                                              [tex]\displaystyle y' = (2x - 5)[20 - 6x + 5][/tex][Addition] Combine like terms:                                                                        [tex]\displaystyle y' = (2x - 5)(25 - 6x)[/tex]Factor:                                                                                                               [tex]\displaystyle y' = -(2x - 5)(6x - 25)[/tex]

Topic: AP Calculus AB/BC (Calculus I/I + II)

Unit: Derivatives

Book: College Calculus 10e

What number is 18% more than 1257 Round your answer to two decimal places needed,

Answers

1,483.26, multiply 1257 by 1.18 to get to your answer

if f(x) = -3x - 5 and g(x) = 4x-2, find (f+g)(x)

Answers

Answer:

(f+g)(x) = x - 7

Step-by-step explanation:

it is simple - when adding 2 functions your simply add both expressions. and the result is then the function of the sum.

the same applies also to other arithmetic operations.

-3x -5 + 4x - 2 = x - 7

5+35367/dyjgtv^*?*vrjn

1)
Ay
X
1
A)
Nin
B) -2
2
C)
D) 2.
2

Answers

Answer:

A) -1/2

Step-by-step explanation:

the slope is defined by the ratio of y/x.

that means that for a change of a certain number of units in x, y changes this number of units.

e.g. a slope of 5/3 means, if x changes by 3 units, y changes by 5.

the graphic shows a line that goes "down" with increasing x.

that means the slope is negative.

that eliminates C and D.

and we see that when x increases by 2 units, y decreases by 1 unit.

that means the slope is -1/2.

16. Using divisibility tests, check whether the number 240720 is divisible by
2, 3, 4, 5, 6, 8, 9, 10 and 11. (Give reason)

Answers

2: it is, because last digit is even
3: it is, because the sum of the digit, 2+4+7+2=15, is divisible by 3
4. it is, because the last 2 digits, 20, is divisible by 4
5. it is, because last digit is 0
6. it is, because it is divisible by 2 and 3
8. it is, because the last 3 digits, 720, is divisible by 8
9. It is not, because the sum of the digits, which is 15, is not divisible by 9
10. It is, because the last digit is 0
11. It is not, because the difference of alternating digits, which is 2+0+2-4-7-0, is not divisible by 11

if marvin bought 490 soda cans and candy bars, where the ratio is 3:4, respectively, how many of each did marvin buy?

Answers

9514 1404 393

Answer:

210 soda cans280 candy bars

Step-by-step explanation:

The 3+4 = 7 ratio units represent the total of 490 snacks, so each ratio unit represents 490/7 = 70 snacks.

The number of soda cans is 3·70 = 210.

The number of candy bars is 4·70 = 280.

Marvin bought 210 soda cans and 280 candy bars.

7. You are mixing some concrete for a home project, and you've calculated according to the
directions that you need six gallons of water for your mix. But your bucket isn't calibrated, so you
don't know how much it holds. On the other hand, you just finished a two-liter bottle of soda. If you
use the bottle to measure your water, how many times will you need to fill it?

Answers

Answer:

11.4 bottles for US gal.

13.6 bottles for Imperial gal.

Step-by-step explanation:

1 US gallon = 3.8 L

6 US Gallons = 6*3.8 L = 22.8 L = 11.4 bottles

1 Imperial gallon = 4.54 L

6 Imperial gallons = 6*4.54 L = 27.24 L = 13.6 bottles.

I got 7 to go :( help plz

Answers

Answer:

I think the second one is the answer ; 2√b

The 2nd one is the answer i’m pretty sure.

Liner equation with a rapid change over time

Answers

Answer:

y=130(2.5) ^(x-1)

Step-by-step explanation:

This should give you an exponential growth curve. I'm assuming this is what you meant by "rapid change over time".

What is 5^-3?
Please I really need someone to explain to me what do you do when there is a negative sign in the exponent. Thank you :)

Answers

1/15

You take out the negative and do the regular equation but turn it into the fraction with one as the numerator and the solution of the exponent as the denominator.

Answer: 1/125 or 0.008 (simplify 1.125 or should I do it?)

Step-by-step explanation:

Well, when there is a negative sign in the exponent you only move the negative exponents. Moving only negative exponents is recommended because using this rule, you multiply two exponents with the same base and add their powers.

Given: x - 8 > -3.



Choose the solution set.

Answers

Answer:

X=-2,-1,0,1,2,3,4,5,6,

Step-by-step explanation:

IF 8 IS LARGER THAN -3 YOU COULD GET THE FOLLOWING ANSWER

Answer:

x>5

Step-by-step explanation:

move -8 to the right side, change the sign and solve:

x > - 3 + 8

x > 5

When a factory operates from 6 AM to 6 PM, its total fuel consumption varies according to the formula f(t)=0.9t^3−0.6t^0.5+10, where t is the time in hours after 6 AM and f(t) is the number of barrels of fuel oil. Step 1 of 3 : How much fuel is consumed by 11 AM? Round your answer to 2 decimal places.

Answers

Step-by-step explanation:

the answer is in the image above

The amount of oil consumed in 5 hours is 120.5 barrels.

What is an equation?

In mathematics, an equation is a formula that expresses the equality of two expressions, by connecting them with the equals sign =.

Given is an equation, of fuel consumption in a factory from 6 am to 6 pm,

The equation =

f(t) = 0.9t³-0.6[tex]t^{0.5}[/tex]+10

where t is the time in hours after 6 am and f(t) is the number of barrels of fuel oil.

We need to find the how much oil is consumed by 11 am,

So, the hours between 11 am to 6 am = 11-6 = 5 hours

So, we need to find the oil consumption in 5 hours, so for the same put t = 5, in the equation,

f(5) = 0.9(5)³-0.6[tex](5)^{0.5[/tex]+10

= 112.5-2+10

= 120.5

Hence, the amount of oil consumed in 5 hours is 120.5 barrels.

Learn more about equations, click;

https://brainly.com/question/29538993

#SPJ2

Suppose you take a survey of all the schools in your state. What would you expect the relationship between the number of students and the number of teachers in each school to be?

Answers

Answer:

25 students:1 teacher

Step-by-step explanation:

in ever class in my school there is 1 teacher and we have between 20 and 30 students therefor 20 + 30 / 2 is 25

Do you guys know this

Answers

Answer:

Kindly check attached picture

Step-by-step explanation:

The expected graph is attached in the picture below :

Since additional fee is charged on only luggage exceeding 50 pound weight :

The inequality is :

Additional fee applied to :

Weight > 50

Hence x our arrows starts from 50 to the right of the number line

what is 72 Times 27 equal

Answers

Answer:

1,944

Step-by-step explanation:

72x27=1,944

Have a wonderful day or night

Answer

1944

Step-by-step explanation:

72*27 = 1944

the image above should clarify how you're actually supposed to do it! couldn't submit this without 20 characters :,)

Which one is this please help thank u

Answers

Answer:

isosceles

Step-by-step explanation:

Two sides are the same length as indicated by the lines on the sides.

This tells use that the triangle is isosceles

Answer:

Hi, this an isoceles triangle

Step-by-step explanation:

An isoceles triangle has 2 equal sides

A right angled one has a right angle (90 degrees)

An equilateral traingle has all of his sides equal

A scalene has all of his sides different.

So this must be an isoceles triangle because there is basically a proof of the two equal sides (the bars).

Hope its clear!

A stamp collection is purchased for $1,000. Twenty years later, the owner is told that the collection is worth quite a bit of money! If the rate of return on the stamp collection is 4% per year, what is the current value of the stamp collection? In your final answer, include all of your calculations.

Answers

Answer:

The current value of the stamp collection is of $2,191.

Step-by-step explanation:

Compound interest:

The compound interest formula is given by:

[tex]A(t) = P(1 + \frac{r}{n})^{nt}[/tex]

Where A(t) is the amount of money after t years, P is the principal(the initial sum of money), r is the interest rate(as a decimal value), n is the number of times that interest is compounded per year and t is the time in years for which the money is invested or borrowed.

A stamp collection is purchased for $1,000.

This means that [tex]P = 1000[/tex]

The rate of return on the stamp collection is 4% per year

This means that [tex]n = 1, r = 0.04[/tex]

So

[tex]A(t) = P(1 + \frac{r}{n})^{nt}[/tex]

[tex]A(t) = 1000(1 + 0.04)^{t}[/tex]

[tex]A(t) = 1000(1.04)^{t}[/tex]

What is the current value of the stamp collection?

This is A(20). So

[tex]A(20) = 1000(1.04)^{20} = 2191[/tex]

The current value of the stamp collection is of $2,191.

Answer:

Step-by-step explanation:

y = 1000(1+0.04)^20

y = 1000(1.04)^20

Rounded to the nearest hundredth

y = 1000(2.19)

y = $2190

The following data represent the maximum wind speed (in knots) and atmospheric pressure (in millibars) for a random sample of hurricanes that originated in the Atlantic Ocean.

Atmospheric Pressure (mb) Wind Speed (knots) Atmospheric Pressure (mb) Wind Speed (knots)
993 50 1006 40
995 60 942 120
994 60 1002 40

Required:
a. Find the y-intercept of the least-squares regression line, treating atmospheric pressure as the explanatory variable (round to four decimal places.)
b. Find the slope of the least-squares regression line, treating atmospheric pressure as the explanatory variable (round to four decimal places.)
c. Is it reasonable to interpret the y-intercept of the least-squares regression line, treating atmospheric pressure as the explanatory variable? Why or why not?

Answers

Answer:

Step-by-step explanation:

X                  Y                 X²                              Y²                  XY

993              50               986049                 2500              49650

995              60               990025                 3600              59700

994              60               988036                  3600              59640

1006             40               1012036                 1600              40240

942             120               887364                 14400              113040

1002             40               1004004                 1600              40080

[tex]\sum X: 5932[/tex]   [tex]\sum Y : 370[/tex]     [tex]\sum X^2 : 5867514[/tex]    [tex]\sum Y^2 = 27300[/tex]    [tex]\sum XY : 362350[/tex]

To determine the regression:

[tex]Mean \ (X) = \dfrac{\sum X }{n} \\ \\ = \dfrac{5932}{6} \\ \\ = 988.67[/tex]

[tex]Mean \ (Y) = \dfrac{\sum Y}{n} \\ \\ = \dfrac{370}{6} \\ \\ = 61.67[/tex]

Intercept [tex]b_o = \dfrac{\sum YX *\sum X^2 - \sum X \sum Y}{n(\sum X^2) - (\sum X)^2}[/tex]

[tex]=\dfrac{370(5867514) -(5932)(370)}{6(5867514) - (5932)^2}[/tex]

= 131760.9563

Slope [tex]b_1 = \dfrac{n(\sum XY) -(\sum X *\sum Y) }{n(\sum X^2)-(\sum X)^2}[/tex]

[tex]b_1 = \dfrac{6(362350) -(5932*370) }{6(5867514)-(5932)^2}[/tex]

[tex]b_1 = -1.2600[/tex]

The regression line equation [tex]Y = b_o +b_1X[/tex]

[tex]Y = 131760.96 -1.2600 X[/tex]

We then make a comparison of the slope of the equation to y = mx+c

slope of the equation = -1.2600

the y-intercept corresponds to when X = 0, thus:

y-intercept = 131760.9563

Yes, it is reasonable to interpret the y-intercept of the regression line, Using atmospheric pressure as an explanatory variable due to the fact that:

X is the independent variable and Y exists as the dependent variable.

mong moi nguoi giup do

Answers

mong moi do giup mguoi

서울기업(주)는 2X20년 7월 5일에 장기간 보유 목적으로 원주기업(주)의 주식 1,200주를 1주당 15,000원에 매수하고 수표를 발행하여 매수대금을 지급하였다. 취득 부대비용은 없다. 이 1,200주는 원주기업(주)이 발행한 주식의12%에 해당하고, 서울기업(주)은 이를 FVOCI 측정 추자주식으로 분류하였다. 서울기업(주)은 원주기업(주) 주식 외에는 다른 회사 주식을 보유하고있지 안다. 다음은 원주기업(주) 주식에 대한 자료이다.

(1) 2X20년 말에 원주기업(주) 주식의 1주당 주가는 14,000원이었다.

(2) 2X20년 3월 25일에 원주기업(주)으로부터 배당금 150만원을 현금으로 받았다.

(3) 2X21년 말에 원주기업(주) 주식의 1주당 주가는 18,000원이었다.

(4) 2X22년 4월 20일에 서울기업(주)은 원주기업(주) 주식을 주당 17,500원에 전부 매각처분하고 처분대금을 현금으로 받았다.

물음: 서울기업(주)의 2X21년 말 재무상태표에 표시되는 장기투자주식 금액과, 기타포괄손익누계액 금액 그리고 포괄손익계산서에 표시되는 기타포괄손익(평가손익)은 각각 얼마인가?

Answers

Answer:

ディーズナッツ

Step-by-step explanation:

You deposit $10,000 in an account that pays 4.5% interest compounded quarterly. Find the future value after one year. ​

Answers

Answer:

11800

Step-by-step explanation:

4.5%=0.045

0.045x10,000=450

450x4(quarterly)=1800

10,000+1800= 11800

Answer:

After 1 year, $10,457.65

Step-by-step explanation:

P = $ 10, 000

r = 4.5% = 0.045

T = 1 year

n = 4 ( compounded quarterly )

      [tex]A = P( 1 + \frac{r}{n})^{nt}[/tex]

         [tex]= 10000( 1 + \frac{0.045}{4})^{4 \times 1}\\\\=10000 \times 1.01125^{4}\\\\= 10000 \times 1.04576508633\\\\= 10457.6508633\\\\= \$ 10, 457.65[/tex]

Other Questions
An excursion group of four is to be drawn from among 5 boys and 6 girls.Find :a. the number of ways of choosing the excursion group if the group :i. Is to be made up of an equal number of boys and girls.ii. Is to be either all-boys or all-girls.III. Has no restriction on its composition.b. Whatis the probability that a random choice of numbers from the group will result in 3 boys and 1 girl. Read the sentence. How many adverbial phrases are included in the sentence? The treasure covered in dust and webs waited beneath the stairs, safely hidden under the mildewing books and tattered old newspapers. one two three four 1. Ano ang basehan mo, ng iyong kamag-anak o maging ng iyong kaibigan sa pagpili ng restawran nakakainan? Question 8 of 10Which question can be answered using the scientific process? 5. In the triangle ABC. M is the midpoint of [AB] and N is the midpoint of [CM].Find the coordinates of N. Given that A(-1, 3), B(7-3) and C(1,6). How many liters does 3 moles of Oxygen gas occupy at standard temperature and pressure A jug is 80% full of water. After 360 millimeters of water are poured out, the jug is 60% filled with water. What is the capacity of the jug?15 POINTS!! Solve the system by substitution.- 2x 10y = -2-4y = x Calculate the mean and the standard deviation of the age of individuals that purchased skateboarding shoes. Use 10 as the midpoint of the first class. (Do not round intermediate calculations. Round your answers to 2 decimal places.) Find the missing term given below. 14,916,2536,? HELP ANYONE OUT THERE WITH MY MATH Which one of the following is an example of dramatic cultural adaptation that impactsglobalization on cultural diversity?A. For an area that is hot and humid, people move to a colder climate.B. For an area that lacks fertile farmland, food is imported.C. For an area that is often windy and cold the common practice is to wear hats,D. For an area that is snowy people move to a warmer climate, please help me ..im begging you Given d= 4t+1 can you determine the value of the y-intercept?HELP!! asap! What is the difference between thecelebration of Independence Day July 4th andJuneteenth? If f(x) = 5x + 3, what is the value of f(-2)? Find the volume of this cylinder._____ cubic inches Can someone please please please help me and show working I am stuck. The figure shows three displacement vectors, which arelabeled a, 5, and c. What is the magnitude and directionof the resultant vector found by adding 5 and ?N4S15 m westt2.0 m east7.0 m eastA. 5.0 m eastB. 5.0 m westC. 9.0 m west0D. 9.0 m east Brains or Brawl? List the reason why you choose what you choose.